Change in angular velocity of the following system

In summary, the pink spheres cause an acceleration of 1 cm per second squared towards themselves. The aqua spheres cause an acceleration of 2 cm per second towards themselves. All spheres have the same mass. The spheres are attached to a massless, yet mysteriously solid perfectly circular green ring, equidistant from the closest spheres (pi/2 from one another). The ring is rotating at a velocity of 1 cm per second. Everything except the pedestal experiences an acceleration of 10 cm per second squared downwards (in the direction from the bearing towards the pedestal). The pedestal is fixed in place (does not accelerate downwards). At what rate does the velocity of the ring change over time if acceleration transmission is instantaneous? At
  • #36
Matt Benesi said:
still looking at the math side of the problem
The math side of the problem starts with using a consistent theory of gravity. This approach is fundamentally wrong. You need a field theory to make it relativistic and it needs to be a tensor field to be consistent.
 
Physics news on Phys.org
  • #37
It looks like you're saying that non-tensorial line element solutions (Schwarzschild and Kerr), which are the only actual confirmations of GR, are not good enough.
 
  • #38
Matt Benesi said:
It looks like you're saying
I never said anything remotely similar.

Matt Benesi said:
non-tensorial line element solutions (Schwarzschild and Kerr),
Those are both tensor solutions.

Matt Benesi said:
which are the only actual confirmations of GR
This isn't true either.

Please stop posting misinformation about physics and please don't misrepresent my comments.
 
  • Like
Likes mfb
  • #39
I found a solution for angular momentum loss due to gravitational waves for 2 bodies. Since I rewrote the math to include #s of bodies other than 4, I can apply the relativistic angular momentum loss equation to the 2 body problem (although a 4 body problem is still a problem).

I'll let you know how it turns out.

So much for that. It's over 10^-70 too small to account for the angular acceleration, unless I totally coded it wrong.
 

Similar threads

Replies
4
Views
8K
  • Introductory Physics Homework Help
Replies
30
Views
2K
Replies
9
Views
1K
Replies
21
Views
17K
Replies
1
Views
968
Replies
1
Views
704
  • Introductory Physics Homework Help
10
Replies
335
Views
8K
Replies
6
Views
1K
Back
Top